Jump to content

Photo

[TOPIC] ÔN THI BẤT ĐẲNG THỨC $\boxed{\text{THPT CHUYÊN}}$ LỚP $10$ năm $2018-2019$

bất đẳng thức holder cosi bunhiacopxki

  • This topic is locked This topic is locked
318 replies to this topic

#21
Korkot

Korkot

    Thượng sĩ

  • Thành viên
  • 227 posts

Bài 12

Cho các số thực dương a,b,c. Chứng minh:

$\frac{a}{b} + \frac{b}{c} + \frac{c}{a} \ge \frac{{a + b + c}}{{\sqrt[3]{{abc}}}}$ (Sưu tầm)

Cm bđt Côsi 3 số trước ( bạn có thể tra google)

Ta có a/b +b/c +c/a>= 3\sqrt[3]{a/b * b/c *c/a}=3

Và a+b+c>= 3sqrt[3]{abc} <=> (a+b+c)/3sqrt[3]{abc} <= 1 

<=> (a+b+c)/sqrt[3]{abc} <= 3

=> đpcm

sorry máy không đánh đươc latex ai giúp mình với

Và xin cho hỏi làm thế nào để post câu hỏi? 


  Nếu bạn cứ tiếp tục ca thán về cùng một nỗi buồn, cùng một việc nhỏ nhặt, bạn sẽ mãi mãi chìm đắm trong thất bại và sống một  cuộc đời nhỏ bé. Hãy luôn nhớ rằng, ngay cả một ngày tồi tệ nhất cũng chỉ có 24 tiếng đồng hồ mà thôi.

                   :like  :like  :like  :like  :like  :like  :like  :like  :like  :like  :like  :like  :like  :like  :like  :like  :like  :like  :like  :like 


#22
Korkot

Korkot

    Thượng sĩ

  • Thành viên
  • 227 posts

Bài 14: 

Tìm GTLN và GTNN của $y=x+\sqrt{12-3x^2}$


Edited by MoMo123, 17-04-2018 - 17:12.

  Nếu bạn cứ tiếp tục ca thán về cùng một nỗi buồn, cùng một việc nhỏ nhặt, bạn sẽ mãi mãi chìm đắm trong thất bại và sống một  cuộc đời nhỏ bé. Hãy luôn nhớ rằng, ngay cả một ngày tồi tệ nhất cũng chỉ có 24 tiếng đồng hồ mà thôi.

                   :like  :like  :like  :like  :like  :like  :like  :like  :like  :like  :like  :like  :like  :like  :like  :like  :like  :like  :like  :like 


#23
dat102

dat102

    Trung sĩ

  • Thành viên
  • 150 posts

Cm bđt Côsi 3 số trước ( bạn có thể tra google)

Ta có a/b +b/c +c/a>= 3\sqrt[3]{a/b * b/c *c/a}=3

Và a+b+c>= 3sqrt[3]{abc} <=> (a+b+c)/3sqrt[3]{abc} <= 1 

<=> (a+b+c)/sqrt[3]{abc} <= 3

=> đpcm

sorry máy không đánh đươc latex ai giúp mình với

Và xin cho hỏi làm thế nào để post câu hỏi? 

Ngược dấu kìa bạn ơi...


:ukliam2:  $\sqrt{MF}$  :ukliam2: 


#24
HelpMeImDying

HelpMeImDying

    Trung sĩ

  • Thành viên
  • 108 posts

Bài 13: Cho a, b, c >0 và a+b+c=1. CMR: $5(a^2+b^2+c^2)\leq 6(a^3+b^3+c^3)+1$

Đẳng thức xảy ra khi nào?

bđt $\Leftrightarrow 6(a^{3}+b^{3}+c^{3})+1\geq 5(a+b+c)(a^{2}+b^{2}+c^{2})\Leftrightarrow a^{3}+b^{3}+c^{3}+(a+b+c)^{3}\geq 5ab(a+b)+5bc(b+c)+5ca(c+a)\Rightarrow 2(a^{3}+b^{3}+c^{3})+3(a+b)(b+c)(c+a)\geq 5ab(a+b)+5bc(b+c)+5ca(c+a)\Leftrightarrow 2(a^{3}+b^{3}+c^{3}+3abc)\geq 2(ab(a+b)+bc(b+c)+ca(c+a))$ (đúng theo bđt Schur)



#25
NHoang1608

NHoang1608

    Sĩ quan

  • Thành viên
  • 375 posts

Lâu ngày a cho 1 bài khá hay nhé :)) 

Bài 15: Cho $x,y,z$ thỏa mãn $0 < x,y,z < 1$ và $xy+yz+zx+2xyz=1$. Tìm giá trị lớn nhất của biểu thức $P= \sqrt{1-x^{2}}+\sqrt{1-y^{2}}+\sqrt{1-z^{2}}$


Edited by MoMo123, 17-04-2018 - 17:07.

The greatest danger for most of us is not that our aim is too high and we miss it, but that it is too low and we reach it.

----- Michelangelo----


#26
hoangkimca2k2

hoangkimca2k2

    Sĩ quan

  • Thành viên
  • 477 posts

$\boxed{\text{Bài 4}}$ Cho $m,n$ là 2 số tự nhiên sao cho $\sqrt{7}-\frac{m}{n}>0$
Chứng minh rằng                                 $\sqrt{7}n-m > \frac{1}{m}$

  :)

$m=1$ thì rõ rồi. Với $m\neq 1$$\Rightarrow m>1$

Ta cần chứng minh $\sqrt{7}n-m > \frac{1}{m}\Leftrightarrow \sqrt{7}n> \frac{1}{m}+m$

$\Leftrightarrow 7n^{2}> m^{2}+\frac{1}{m^{2}}+2$

Lại có: $7n^{2}-m^{2}\geq 3 >2+\frac{1}{m^{2}}$


  N.D.P 

#27
hoangkimca2k2

hoangkimca2k2

    Sĩ quan

  • Thành viên
  • 477 posts

Bài 16: Với $a,b,c>0$ và $a+b+c=1$, $a,b,c< \frac{1}{2}$. Chứng minh rằng $\frac{1-2a^{2}}{(1-2a)^{2}}+\frac{1-2b^{2}}{(1-2b)^{2}}+\frac{1-2c^{2}}{(1-2c)^{2}}\geq 21$


Edited by MoMo123, 17-04-2018 - 21:13.

  N.D.P 

#28
hoangkimca2k2

hoangkimca2k2

    Sĩ quan

  • Thành viên
  • 477 posts

Bài 17: Cho $a,b,c>1$ và $a+b+c=abc$. tìm giá trị nhỏ nhất của biểu thức

$P= \frac{b-2}{a^{2}}+\frac{c-2}{b^{2}}+\frac{a-2}{c^{2}}$


Edited by MoMo123, 17-04-2018 - 21:25.

  N.D.P 

#29
Korkot

Korkot

    Thượng sĩ

  • Thành viên
  • 227 posts

Ngược dấu kìa bạn ơi...

Ở mẫu phải đổi dấu mà?


  Nếu bạn cứ tiếp tục ca thán về cùng một nỗi buồn, cùng một việc nhỏ nhặt, bạn sẽ mãi mãi chìm đắm trong thất bại và sống một  cuộc đời nhỏ bé. Hãy luôn nhớ rằng, ngay cả một ngày tồi tệ nhất cũng chỉ có 24 tiếng đồng hồ mà thôi.

                   :like  :like  :like  :like  :like  :like  :like  :like  :like  :like  :like  :like  :like  :like  :like  :like  :like  :like  :like  :like 


#30
HelpMeImDying

HelpMeImDying

    Trung sĩ

  • Thành viên
  • 108 posts

Lâu ngày a cho 1 bài khá hay nhé :)) 

Bài 15: Cho $x,y,z$ thỏa mãn $0 < x,y,z < 1$ và $xy+yz+zx+2xyz=1$. Tìm giá trị lớn nhất của biểu thức $P= \sqrt{1-x^{2}}+\sqrt{1-y^{2}}+\sqrt{1-z^{2}}$

$P\leq \sqrt{3(3-x^{2}-y^{2}-z^{2})}$

Từ gt: $1=xy+yz+zx+2\sqrt{x^{2}y^{2}z^{2}}\leq xy+yz+zx+\frac{2(xy+yz+zx)\sqrt{xy+yz+zx}}{3\sqrt{3}}$

Đặt $\frac{\sqrt{xy+yz+zx}}{\sqrt{3}}=t$$\Rightarrow 2t^{3}+3t^{2}\geq 1\Leftrightarrow (t+1)^{3}(2t-1)\geq 0\Leftrightarrow t\geq \frac{1}{2}\Rightarrow xy+yz+zx\geq \frac{3}{4}$

$\Rightarrow x^{2}+y^{2}+z^{2}\geq xy+yz+zx\geq \frac{3}{4}\Rightarrow P\leq \sqrt{3(3-\frac{3}{4})}= \frac{3\sqrt{3}}{2}$



#31
hoangkimca2k2

hoangkimca2k2

    Sĩ quan

  • Thành viên
  • 477 posts

$\boxed{\text{Bài 3}}$ Cho các số thực x,y,z thỏa mãn $\left\{\begin{matrix}0\leq x\leq y\leq z & & \\ x+y+z=3 & & \end{matrix}\right.$
Tìm  Max                         $ P = \frac{x}{y^3+16} +\frac{y}{z^3 +16} +\frac{z}{x^3+16}$


 

 

Ta có: $0\leq x\leq y\leq z$. Khi đó $P\leq \frac{x}{x^{3}+16}+\frac{y}{x^{3}+16}+\frac{z}{x^{3}+16}$

$=\frac{3}{x^{3}+16}$ (vì $x+y+z=3$)$\leq \frac{3}{16}$ ( vì $x\geq 0$)


Edited by hoangkimca2k2, 17-04-2018 - 09:57.

  N.D.P 

#32
hoangkimca2k2

hoangkimca2k2

    Sĩ quan

  • Thành viên
  • 477 posts

Bài 18: Cho $a,b,c$ là các số thực dương. Chứng minh rằng: 

$\frac{a}{\sqrt{b+c}}+\frac{b}{\sqrt{a+c}}+\frac{c}{\sqrt{a+b}}\geq \frac{1}{\sqrt{2}}(\sqrt{a}+\sqrt{b}+\sqrt{c})$


Edited by MoMo123, 17-04-2018 - 23:23.

  N.D.P 

#33
tr2512

tr2512

    Thượng sĩ

  • Thành viên
  • 272 posts

Bài 19:

Cho các số thực không âm a, b, c. Chứng minh bất đẳng thức:

$\frac{1}{{{{\left( {a + b} \right)}^2}}} + \frac{1}{{{{\left( {b + c} \right)}^2}}} + \frac{1}{{{{\left( {a + c} \right)}^2}}} \ge \frac{9}{{4\left( {{a^2} + {b^2} + {c^2}} \right)}}$

 (Sưu tầm)

P/s: bài này đừng giải theo kiểu Iran 1996 :D


Edited by MoMo123, 17-04-2018 - 23:20.


#34
tr2512

tr2512

    Thượng sĩ

  • Thành viên
  • 272 posts

Bài 14: 

Tìm GTLN và GTNN của y=x+sqrt[2]{12-3x^2}

Bạn nên gõ bằng Latex



#35
Korkot

Korkot

    Thượng sĩ

  • Thành viên
  • 227 posts

Bạn nên gõ bằng Latex

Máy gõ Latex không được . Mình thử nhiều rồi


  Nếu bạn cứ tiếp tục ca thán về cùng một nỗi buồn, cùng một việc nhỏ nhặt, bạn sẽ mãi mãi chìm đắm trong thất bại và sống một  cuộc đời nhỏ bé. Hãy luôn nhớ rằng, ngay cả một ngày tồi tệ nhất cũng chỉ có 24 tiếng đồng hồ mà thôi.

                   :like  :like  :like  :like  :like  :like  :like  :like  :like  :like  :like  :like  :like  :like  :like  :like  :like  :like  :like  :like 


#36
thanhdatqv2003

thanhdatqv2003

    Trung sĩ

  • Thành viên
  • 159 posts

Ta có: $0\leq x\leq y\leq z$. Khi đó $P\leq \frac{x}{x^{3}+16}+\frac{y}{x^{3}+16}+\frac{z}{x^{3}+16}$

$=\frac{3}{x^{3}+16}$ (vì $x+y+z=3$)$\leq \frac{3}{16}$ ( vì $x\geq 0$)

anh ơi rua dau = xay ra khi x=y=z=0 roi


:ohmy: [Không tồn tại các nghiệm nguyên khác không x, y, và z thoả mãn xn + yn = zn trong đó n là một số nguyên lớn hơn 2.  (FERMAT)  :ohmy: 

 

 

 

 


#37
hoangkimca2k2

hoangkimca2k2

    Sĩ quan

  • Thành viên
  • 477 posts

anh ơi rua dau = xay ra khi x=y=z=0 roi

Dấu bằng xảy ra khi $x=y=0$ và $z=3$


  N.D.P 

#38
thanhdatqv2003

thanhdatqv2003

    Trung sĩ

  • Thành viên
  • 159 posts

  

Lại có: $7n^{2}-m^{2}\geq 3 >2+\frac{1}{m^{2}}$

tai sao lai >= 3 vay anh


:ohmy: [Không tồn tại các nghiệm nguyên khác không x, y, và z thoả mãn xn + yn = zn trong đó n là một số nguyên lớn hơn 2.  (FERMAT)  :ohmy: 

 

 

 

 


#39
thanhdatqv2003

thanhdatqv2003

    Trung sĩ

  • Thành viên
  • 159 posts

Dấu bằng xảy ra khi $x=y=0$ và $z=3$

mà đoan đau ta co P =< .....  thi dau = xay ra khi x=y=z maf


:ohmy: [Không tồn tại các nghiệm nguyên khác không x, y, và z thoả mãn xn + yn = zn trong đó n là một số nguyên lớn hơn 2.  (FERMAT)  :ohmy: 

 

 

 

 


#40
Hoang Dinh Nhat

Hoang Dinh Nhat

    Sĩ quan

  • Thành viên
  • 402 posts

A cũng góp một bài khá hay: 

Bài 20: Cho $a,b,c,x,y,z>0$ thỏa mãn $(a+b+c)(x+y+z)=(a^2+b^2+c^2)(x^2+y^2+z^2)=4$. Chứng minh: $abcxyz<\frac{1}{36}$

Ps: Lâu lắm rồi mới vào lại diễn đàn


Edited by MoMo123, 17-04-2018 - 17:10.

Chấp nhận giới hạn của bản thân, nhưng đừng bao giờ bỏ cuộc

 

 

 

 






Also tagged with one or more of these keywords: bất đẳng thức, holder, cosi, bunhiacopxki

1 user(s) are reading this topic

0 members, 1 guests, 0 anonymous users